"Lagrangian formulation of electromagetism"의 두 판 사이의 차이

수학노트
둘러보기로 가기 검색하러 가기
imported>Pythagoras0
imported>Pythagoras0
19번째 줄: 19번째 줄:
 
$$
 
$$
 
* 전하가 받는 힘 $\mathbf{F}$는 다음과 같다
 
* 전하가 받는 힘 $\mathbf{F}$는 다음과 같다
:<math>\mathbf{F}=e\mathbf{E}+e\mathbf{v}\times \mathbf{B}</math>
+
:<math>\mathbf{F}=e(\mathbf{E}+\mathbf{v}\times \mathbf{B})</math>
* 이 중에서 $e\mathbf{v}\times \mathbf{B}$를 로렌츠 힘이라 한다
+
* 이를 로렌츠 힘이라 한다
 
* http://en.wikipedia.org/wiki/Lorentz_force
 
* http://en.wikipedia.org/wiki/Lorentz_force
 
* {{수학노트|url=전자기_텐서와_맥스웰_방정식}}
 
* {{수학노트|url=전자기_텐서와_맥스웰_방정식}}
26번째 줄: 26번째 줄:
  
 
==전자기장에 대한 라그랑지안==
 
==전자기장에 대한 라그랑지안==
===free===
+
===상호작용이 없는 경우===
* 상호작용이 없는 전자기장의 라그랑지안은 다음과 같다
+
* $j$와 $\rho$가 0인 경우
 +
* 전자기장에 대한 라그랑지안은 다음과 같다
 
$$\mathcal{L}_{\text{EM}}= - \frac{1}{4}F_{\mu\nu}F^{\mu\nu}=\frac{1}{2}(\mathbf{E}^2-\mathbf{B}^2)$$
 
$$\mathcal{L}_{\text{EM}}= - \frac{1}{4}F_{\mu\nu}F^{\mu\nu}=\frac{1}{2}(\mathbf{E}^2-\mathbf{B}^2)$$
 
이 때 <math>F_{\mu\nu} = \partial_\mu A_\nu - \partial_\nu A_\mu \,\!</math>는 전자기텐서, $A=(A_{\mu})$는 전자기 포텐셜
 
이 때 <math>F_{\mu\nu} = \partial_\mu A_\nu - \partial_\nu A_\mu \,\!</math>는 전자기텐서, $A=(A_{\mu})$는 전자기 포텐셜
* action
+
* 작용
 
:<math>S=-\frac{1}{4}\int F^{\alpha\beta}F_{\alpha\beta}\,d^{4}x</math>
 
:<math>S=-\frac{1}{4}\int F^{\alpha\beta}F_{\alpha\beta}\,d^{4}x</math>
 
* 라그랑지안은 전자기 포텐셜의 다음과 같은 변환에 대하여 불변이다
 
* 라그랑지안은 전자기 포텐셜의 다음과 같은 변환에 대하여 불변이다
 
:<math>A_{\mu}(x) \to A_{\mu}(x)-\partial_{\mu}\Lambda(x)</math>
 
:<math>A_{\mu}(x) \to A_{\mu}(x)-\partial_{\mu}\Lambda(x)</math>
 
여기서 $\Lambda(x)$는 임의의 스칼라장
 
여기서 $\Lambda(x)$는 임의의 스칼라장
* equation of motion
+
* 운동방정식
 
$$
 
$$
 
\partial_\mu F^{\mu\nu}=0
 
\partial_\mu F^{\mu\nu}=0
 
$$
 
$$
* see http://www.damtp.cam.ac.uk/user/tong/qft/six.pdf
 
  
===in the presence of $j$ and $\rho$===
+
===상호작용이 있는 경우===
* Lagrangian
+
* $j$$\rho$가 0이 아닌 경우
 +
* 전자기장에 대한 라그랑지안은 다음과 같다
 
$$L=-\frac{1}{4}F_{\mu\nu}F^{\mu\nu}-ej_\mu A^\mu$$
 
$$L=-\frac{1}{4}F_{\mu\nu}F^{\mu\nu}-ej_\mu A^\mu$$
* action
+
* 작용
 
$$S[\phi,A]=\int_{t_1}^{t_2}\int_{\mathbb{R}^3}\left(-\rho\phi+j\cdot A+\frac{\epsilon_0}{2}E^2-\frac{1}{2\mu_0}B^2\right)\,dV\,dt$$
 
$$S[\phi,A]=\int_{t_1}^{t_2}\int_{\mathbb{R}^3}\left(-\rho\phi+j\cdot A+\frac{\epsilon_0}{2}E^2-\frac{1}{2\mu_0}B^2\right)\,dV\,dt$$
* w.r.t $\phi$
+
* 운동방정식
$$\nabla\cdot E=\frac{\rho}{\epsilon_0}$$
+
$$
* w.r.t $A$
+
\nabla\cdot E=\frac{\rho}{\epsilon_0}\\
$$\nabla\times B=\mu_0j+\epsilon_0\mu_0\frac{\partial E}{\partial t}$$
+
\nabla\times B=\mu_0j+\epsilon_0\mu_0\frac{\partial E}{\partial t}
 +
$$
  
  
55번째 줄: 57번째 줄:
  
 
==memo==
 
==memo==
* http://dexterstory.tistory.com/888
+
* http://dexterstory.tistory.com/724
 
 
  
  
73번째 줄: 74번째 줄:
 
* Susskind, [http://www.lecture-notes.co.uk/susskind/classical-mechanics/lecture-8/the-electromagnetic-lagrangian/ The electromagnetic Lagrangian]
 
* Susskind, [http://www.lecture-notes.co.uk/susskind/classical-mechanics/lecture-8/the-electromagnetic-lagrangian/ The electromagnetic Lagrangian]
 
* Lecture 8 | Modern Physics: Classical Mechanics (Stanford). 2008. http://www.youtube.com/watch?v=gUUbl444r74&feature=youtube_gdata_player.
 
* Lecture 8 | Modern Physics: Classical Mechanics (Stanford). 2008. http://www.youtube.com/watch?v=gUUbl444r74&feature=youtube_gdata_player.
* Higgs mechanism
+
* http://www.damtp.cam.ac.uk/user/tong/qft/six.pdf
** http://unapologetic.wordpress.com/2012/07/16/the-higgs-mechanism-part-1-lagrangians/
 
** http://unapologetic.wordpress.com/2012/07/17/the-higgs-mechanism-part-2-examples-of-lagrangian-field-equations/
 
** http://unapologetic.wordpress.com/2012/07/18/the-higgs-mechanism-part-3-gauge-symmetries/
 
** http://unapologetic.wordpress.com/2012/07/19/the-higgs-mechanism-part-4-symmetry-breaking/
 
  
  
 
==questions==
 
==questions==
 
* http://physics.stackexchange.com/questions/3005/derivation-of-maxwells-equations-from-field-tensor-lagrangian?rq=1
 
* http://physics.stackexchange.com/questions/3005/derivation-of-maxwells-equations-from-field-tensor-lagrangian?rq=1

2014년 1월 29일 (수) 06:48 판

하전입자에 대한 라그랑지안

  • 전기장 $\mathbf{E}=\nabla \phi$
  • 자기장 $\mathbf{B}=\nabla \times \mathbf{A}$
  • 전자기장 안에 놓인 질량 $m$, 전하 $e$의 하전입자에 대한 라그랑지안

\[L(q,\dot{q})=\frac{m||\dot{q}||^2}{2}-e\phi+eA_{i}\dot{q}^{i}\]

  • 켤레운동량

\[p_{i}=\frac{\partial{L}}{\partial{\dot{q}^{i}}}=m \dot{q}_{i}+eA_{i}=mv_{i}+eA_{i}\]

  • 오일러-라그랑지 방정식 \(\dot{p}=F\)은 다음과 같이 쓰여진다

$$ \dot{p}_{i}=m\frac{dv_{i}}{dt}+e\frac{\partial{A_{i}}}{\partial t}+e\frac{\partial{A_{i}}}{\partial{q}^{j}}\dot{q}^{j}=m\frac{dv_{i}}{dt}+e\frac{\partial{A_{i}}}{\partial{q}^{j}}\dot{q}^{j} \\ F_{i}=\frac{\partial{L}}{\partial{q^{i}}}=\frac{\partial}{\partial{{q}^{i}}}(-e\phi+eA_{j}\dot{q}^{j})=-e\frac{\partial{\phi}}{\partial{q}^{i}} +e\frac{\partial{A_{j}}}{\partial{q}^{i}}\dot{q}^{j} $$ \[m\frac{dv_{i}}{dt}=eE_{i}+eF_{ij}\dot{q}^{j},\quad i=1,2,3 \label{eom}\] 여기서 $F_{\mu\nu} = \partial_\mu A_\nu - \partial_\nu A_\mu \,\!$

  • $F_{12}=B_{3}$, $F_{23}=B_{1}$, $F_{31}=B_{2}$
  • 가령 $i=1$이면, \ref{eom}은 다음과 같다

$$ ma_1=eE_1+e(F_{11}\dot{q}^{1}+F_{12}\dot{q}^{2}+F_{13}\dot{q}^{3})=eE_1+e(F_{12}\dot{q}^{2}-F_{31}\dot{q}^{3})=eE_1+e(\mathbf{v}\times \mathbf{B})_{1} $$

  • 전하가 받는 힘 $\mathbf{F}$는 다음과 같다

\[\mathbf{F}=e(\mathbf{E}+\mathbf{v}\times \mathbf{B})\]


전자기장에 대한 라그랑지안

상호작용이 없는 경우

  • $j$와 $\rho$가 0인 경우
  • 전자기장에 대한 라그랑지안은 다음과 같다

$$\mathcal{L}_{\text{EM}}= - \frac{1}{4}F_{\mu\nu}F^{\mu\nu}=\frac{1}{2}(\mathbf{E}^2-\mathbf{B}^2)$$ 이 때 \(F_{\mu\nu} = \partial_\mu A_\nu - \partial_\nu A_\mu \,\!\)는 전자기텐서, $A=(A_{\mu})$는 전자기 포텐셜

  • 작용

\[S=-\frac{1}{4}\int F^{\alpha\beta}F_{\alpha\beta}\,d^{4}x\]

  • 라그랑지안은 전자기 포텐셜의 다음과 같은 변환에 대하여 불변이다

\[A_{\mu}(x) \to A_{\mu}(x)-\partial_{\mu}\Lambda(x)\] 여기서 $\Lambda(x)$는 임의의 스칼라장

  • 운동방정식

$$ \partial_\mu F^{\mu\nu}=0 $$

상호작용이 있는 경우

  • $j$와 $\rho$가 0이 아닌 경우
  • 전자기장에 대한 라그랑지안은 다음과 같다

$$L=-\frac{1}{4}F_{\mu\nu}F^{\mu\nu}-ej_\mu A^\mu$$

  • 작용

$$S[\phi,A]=\int_{t_1}^{t_2}\int_{\mathbb{R}^3}\left(-\rho\phi+j\cdot A+\frac{\epsilon_0}{2}E^2-\frac{1}{2\mu_0}B^2\right)\,dV\,dt$$

  • 운동방정식

$$ \nabla\cdot E=\frac{\rho}{\epsilon_0}\\ \nabla\times B=\mu_0j+\epsilon_0\mu_0\frac{\partial E}{\partial t} $$



memo


related items


매스매티카 파일 및 계산 리소스


리뷰, 에세이, 강의노트


questions